You are on page 1of 10

QUANT QUESTIONS THAT HAVE APPEARED IN

CAT IN THE LAST 3 YEARS

Permutations and Combinations 5. For any N, the number of non-mates of each


member of X is
Directions for question 1: Select the correct 1 2
(1) (N − 3N + 2) (2) 2N − 5
alternative from the given choices. 2
1 2
1. X = {0, 1, 2, 3, 5} (3) 3N − 10 (4) (N − 5N + 6)
2
I is the set of integers greater than 999 and not
1 2
exceeding 5000, formed by using one or more (5) (N − 7N + 16)
elements of X as its digits. Find the number of 2
elements in I.
(1) 376 (2) 375 (3) 500 6. For any N, the number of common mates of
(4) 499 (5) 501 two mates in X is
(1) N − 2 (2) 3N − 8
Directions for questions 2 and 3: Answer these 1 2
(3) (N − 6N + 12) (4) N − 1
questions based on the following information. 2
1 2
The figure below is the plan of a town T. The streets (5) (N − 3N)
2
are at right angles to each other. Shoba is a resident
of T whose house is at H and whose office is at O.
Directions for questions 7 to 42: Select the correct
R is a rectangular park in which there is a road along
alternative from the given choices.
the diagonal AB and P is a prohibited region in T.
H C 7. How many five-digit numbers divisible by ‘9’ can
be formed using the digits 0, 1, 2, 4, 6, 8, 9 such
that each digit occurs atmost once in any such
A number?
P
R (A) 96 (B) 192 (C) 288 (D) 144
B 8. How many five-digit numbers divisible by 4 can
be formed using the digits 0, 7, 5, 2, 4, 6 without
repeating any digit?
O (A) 126 (B) 186 (C) 204 (D) 180
2. Shoba wants to go from H to O taking the shortest
path. How many paths can she choose from? 9. Six foot ball players, seven hockey players, eight
(1) 60 (2) 108 (3) 90 tennis players and nine cricket players are to be
(4) 126 (5) 72 arranged in a row. In how many can they be
arranged such that all the players who play the
3. Shoba wants to go from H to the club C, shown in same game sit together?
the figure through O, taking the shortest path. (A) 6!7!8!9! (B) 4!
How many paths can she choose from? (C) 30! (D) 4!6!7!8!9!
(1) 1638 (2) 936 (3) 1170
10. In how many ways can seven students be sent
(4) 1404 (5) 1560
into five different sections?
5
(A) 7 (B) 57
Directions for question 4: Select the correct
(C) 35 (D) None of these
alternative from the given choices.
11. The sum of all the five-digit numbers that can be
K = (p + q + r) . Find the number of distinct terms
15
4. formed by using the digits 2, 4, 6, 7, 8 without
in the expansion of the bracket. repetition is
(1) 136 (2) 153 (3) 145 (A) 7919982 (B) 7199928
(4) 120 (5) 128 (C) 7999182 (D) 7919928
Directions for questions 5 and 6: Answer these 12. There are 40 lines in a plane of which a set of
questions based on the information below. 12 lines are concurrent at A, another set of
15 lines are concurrent at B and the set of
X is the set of all pairs (q, p) where p and q satisfy remaining lines are parallel. What is the number
N ≥ p > q ≥ 1, where N ≥ 4. If any two distinct members of points of intersection of these 40 lines, given
in X have one number in common, they are called that the three sets are disjoint?
“mates”. Otherwise they are called “non-mates”. For (A) 611 (B) 531 (C) 533 (D) 638
example if N = 4, then X = {(3, 4), (2, 3), (2, 4), (1, 2),
(1, 3), (1, 4)}. In X, (1, 3) and (1, 4) are mates (2, 3) 13. The coefficients of how many terms in the
and (3, 4) are also mates but (3, 4) and (1, 2) are expansion of (x + y – z)100 are negative?
non-mates. (A) 2450 (B) 2500 (C) 2550 (D) 2600
1
Geometry & Mensuration 21. A large number (greater than 1200) of equilateral
triangles of side 2 units are available to form
14. PQR is a triangle. PQ = 19.5 cm and QR = 11 cm. equilateral triangles. Equilateral triangles of sides
PS is the altitude of ∆ PQR of length 5 cm. C is a 16 units, 18 units, …… 30 units are formed using
circle circumscribing the triangle PQR. Find the the available triangles. Find the total number of
radius (in cm) of C. triangles used.
(1) 18.05 (2) 28.85 (3) 27.25 (A) 1050 (B) 1100 (C) 1150 (D) 1200
(4) 21.45 (5) 31.25
22. Find the side of the smallest equilateral triangle
15. ABC is an obtuse angled triangle whose sides
are 7 cm, 16 cm and y cm where y is an integer. that can contain a semicircle of radius 8 3 cm.
Find the number of possible values of y. (in cm)
(1) 9 (2) 10 (3) 6 (4) 16 (5) 17 (A) 16 3 (B) 32 (C) 32 3 (D) 40
16. PQRS is a square. M and N are the midpoints of
23. In the figure below, ∆ABC is
PS and QR respectively. X and Y are points lying
equilateral. Three identical
on the line joining M and N inside PQRS, such
circles are shown in it. Each
that ∠PXS = ∠QYR = 60°. Find the ratio of the
circle touches the other two
area of the hexagon PQXRSY and the remaining
circles and has two of the
area inside PQRS.
sides of the triangle being
(1) 4 − 3 (2) 2 3 + 3 (3) 2 3 − 1 tangents to it. Find the ratio of the side of the
(4) 1 + 3 +1 triangle and the radius of each circle.
( ) ( )
3 (5)
(A) 2 2 + 3 (B) 2 1 + 2 3
2(1 + 3 ) 2(1 + 3 3 )
17. Each of two circles passes through the centre of
(C) (D)
the other. The radius of either circle is 2 cm.
Find the area of the intersecting region (in sq.cm).
4π 8π 24. N points are marked on the circumference of a
(1) − 3 (2) +2 3 circle. The number of triangles which can be
3 3 drawn by using these points as vertices is 210
8π 8π more than the number of triangles which can be
(3) −2 3 (4) + 3
3 3 drawn by using all but one of these points as
vertices. Find N.
2π 3
(5) − (A) 21 (B) 22 (C) 23 (D) 24
3 2
25. Let X denote the product of the sides of a triangle.
18. The radius of the base of a right circular cone is
Let Y denote the product of its semi-perimeter,
6 cm and the height is 21 cm. The cylinder having
the maximum possible total surface area is Y
circumradius and its inradius. =
placed inside the cone such that one of its flat X
surfaces rests on the base of the cone. Find the 1 1
(A) (B)
total surface area of the cylinder, in sq.cm. 4 6
1353π 1321π 1383π 1
(1) (2) (3) (C) (D) Cannot be determined
8 8 8 3
1473π 441π
(4) (5) T Q
8 5 26. In the figure above, PQRS P
is a square. A sector of a
19. C1 and C2 are two equal circles whose centers V
circle with centre S is U
are M and N respectively. They intersect at X and shown. TQVU is a
Y. Neither of the centers lies inside the other rectangle such that TU = 9
circle. If ∠XMN = θ, what is the range of possible and UV = 32. Find PS.
value of θ? (A) 60
S R
(1) 0° ≤ θ ≤ 30° (2) 0° ≤ θ ≤ 45° (B) 65
(3) 0° ≤ θ ≤ 60° (4) 0° ≤ θ ≤ 75° (C) 70
(5) 0° ≤ θ ≤ 90° (D) Cannot be determined
20. Two identical circles intersect each other. 27. Two intersecting circles are said to be orthogonal,
The radii of the circles and the distance joining if a pair of tangents drawn to them at any point of
the centres of both the circles are in the ratio of intersection are perpendicular to each other.
1 : 3 . Find the ratio of the area of the region C1 and C2 are orthogonal circles. Each of them
common to both the circles, to that of a circle. has a radius of 8 cm. Find the area of the region
common to both the circles. (in sq cm)
π− 3 3π − 2 3 (A) 16(2π − 1) (B) 16(2π − 3)
(A) (B)
4π 12π (C) 32(π − 1) (D) 32(π − 2)
π− 3 2π − 3 3
(C) (D)
6π 6π
2
28. On the periphery of a square grass field ABCD, Equations, Ratio, Proportion, Variation
two poles are fixed. One of them is fixed at the
midpoint of side AB while the other one is fixed at 36. Amar, Bhuvan, Chetan and Dinesh are
the midpoint of side BC. Side of ABCD is 16 m. four friends. Amar has m marbles with him.
A cow is tied to one of the poles and another is He gives Bhuvan 1 less than half the number of
tied to the other pole. If each cow is tied with a marbles he has. Then he gives Chetan 1 less
8 m long rope, then find the ungrazable area in than half the remaining number of marbles he
the field. (in sq. m) has. Finally, Amar gives Dinesh 1 less than half
(A) 256 − 32π (B) 216 − 32π the remaining number of marbles he has and is
(C) 192 − 32π (D) 240 − 32π left with 4 marbles. Which of the following best
describes the value of m?
29. PQR is an isosceles triangle with PQ = QR. QA is (1) 1 ≤ m ≤ 4 (2) 5 ≤ m ≤ 9
the median to PR. B is a point on QR (or QR (3) 9 ≤ m ≤ 13 (4) 10 ≤ m ≤ 14
extended) such that AQ = AB. ∠PQA = 30°. (5) m ≥ 14
Find ∠ABQ.
(A) 20° (B) 30° 1
37. If a ∝ and b ∝ c , then find the value of
2
(C) 40° (D) Cannot be determined
b
30. A triangle has an area of 960 sq m. Two of its 9a + 5c, when b = 27. Given that when b = 3 then
sides are 68 m and 32 m. Find its third side if it is a = 4 and c = 6.
given that the triangle is right angled (in m). (A) 94 (B) 49 (C) 76 (D) 38
(A) 52 (B) 58
(C) 62 (D) None of these 38. If 3a + 5b + 7c = 1.25 k and 2a + b + 3c = 0.75 k,
then 7b + 5c is what percentage of k?
31. A right circular cone has a radius of 18 cm and a (A) 25% (B) 50% (C) 35% (D) 75%
height of 54 cm. From it, a right circular cylinder
of radius 15 cm was cut. Find the volume of the 39. There are some two rupee coins and five rupee
conical part of the remaining solid (in cubic cm). coins in a bag. If the number of five rupee coins is
(A) 4540π (B) 4375π (C) 4780π (D) 3375π tripled, then the amount in the bag is increased
by 75%. Which of the following can be the
32. Two plastic cubes each of which have an integral number of five rupees coins in the bag?
side (in cm), have the sum of their lateral surface (A) 13 (B) 20 (C) 18 (D) 32
areas equal to 468 sq cm. The sum of their
volumes is 945 cubic cm. Find the volume of the 40. There are 100 questions in a test paper. Four marks
hemisphere whose radius equals the sum of the are awarded for each right answer and two marks
edges of the two cubes (in cubic cm) are deducted for each wrong answer. If Abhilash
(A) 2150π (B) 2250π (C) 2325π (D) 2400π attempts more than 85 questions and get
70 marks, What is the minimum number of
33. In ∆ABC, AB = 32, BC = 24 and AC = 40. Using questions that he could have answered correctly?
B as a centre, a circle is drawn. This circle cuts (A) 40 (B) 38 (C) 39 (D) 41
AB and BC at D and E respectively. The radius of
this circle equals the circumradius of ∆ ABC. 41. Arun, Varun and Kiran have a total of Rs.8,000
Find AD : EC. with them. They spend Rs.50, Rs.100 and
(A) 3 : 1 (B) 2 : 1 (C) 3 : 2 (D) 5 : 2 Rs.200 respectively, Now they have a money is
the ratio of 14 : 22 : 15. What is the amount with
Special Equations the Varun initially?
(A) 2150 (B) 3400
34. In a certain country, the currency used was called (C) 3300 (D) 2100
femto. Raju had his lunch in one of the hotels in it.
His bill came to 47 femtos. He had notes in only Time and Distance
3 denominations – 1-femto, 2-femto and 20-femto.
In how many ways can he settle the bill? 42. P and Q are 2 stations. Raju plans to drive from
(1) 24 (2) 18 (3) 48 (4) 16 (5) 42 city R, located 840 km directly to the north of Q,
210
to Q, at a speed of kmph so that he can
35. A bank teller was making the payment for a 3
cheque presented by Mohan. As he was in a catch a train arriving there from P. The train
confused state of mind, he transposed the rupees leaves P at 9 a.m. and travels at a speed of
and paise and hence gave more than what he
should have. Mohan left the bank and bought a 84 3 kmph. P is between east and south-east
biscuit from a nearby store for Rs.1.50. of R with RQ at 60° to RP. Also P is between the
The amount remaining with him was 4 times the north and the north-east of Q with PQ at 30° to
amount on the cheque. The amount remaining RQ. If Raju has to reach P at least 25 minutes
with him must have been between before the train, then find the approximate latest
(1) Rs.70 and 71 (2) Rs.79 and Rs.80 time at which he should start from R.
(3) Rs.85 and Rs.94 (4) Rs.93 and Rs.94 (1) 6:50 a.m. (2) 6:40 a.m. (3) 6:20 a.m.
(5) Rs.98 and Rs.99 (4) 6:30 a.m. (5) 7:00 a.m.
3
Directions for questions 43 and 44: Answer these result is found to be the sum of the original
questions based on the information below. integers. The least of the integers is denoted by
L. Which of the following holds true?
Cities P and Q are 4800 km apart. P is to the west of (1) 1 ≤ L ≤ 4 (2) 5 ≤ L ≤ 8
Q. Both cities are in different time zones “Best (3) 9 ≤ L ≤ 12 (4) 13 ≤ L ≤ 15
Airlines” was an airline which operates non-stop (5) L > 15
flights between the cities. All its planes cruise at the
same speed in both directions. However, the effective 49. The roots of the equation x3 − px2 + qx − r = 0 are
speed of any plane is influenced by a steady wind a, b and c, which are consecutive integers.
blowing from west to east at 100 kmph. The table
Find the least possible value of q.
below shows the departure time of the planes from
each city and their arrival time at the other city. (1) 0 (2) 1 (3) 2
(4) −1 (5) − 2
(Given below are local times of the respective cities)
Directions for questions 50 and 51: Answer these
Departure Arrival questions using the information below.
City Time City Time
Let g(x) = px + qx + r, where p, q and r are constants
2
P 7:00 a.m. Q 5:00 p.m.
Q 6:00 p.m. P 4.00 a.m. and r ≠ 0. One root of g(x) = 0 is 4 and 8g(3) = –3g(6).

43. Find the time in P when the plane landed in Q. 50. Find the other root of g(x) = 0.
(1) 4:00 p.m. (2) 3:00 p.m. (1) 1 (2) –3 (3) –6
(3) 3:30 p.m. (4) 2:30 p.m. (4) 2 (5) Cannot be determined
(5) Cannot be determined
51. Find the sum of p, q and r.
44. Find the cruising speed of the plane (in kmph). (1) −21 (2) –20 (3) 19
(1) 600 (2) 550 (3) 500 (4) 18 (5) Cannot be determined
(4) 450 (5) Cannot be determined
Directions for questions 52 and 53: Answer these
Directions for questions 45 to 49: Select the questions based on the given information.
correct alternative from the given choices.
Raju makes and sells an item in a market every day.
He sells each unit of that item at Rs.106. The cost of
45. A starts from point P at 9:00 a.m. and travels east
producing x units per day is 200 + px + qx , where p
2
at 45 km/hr. B starts a bit later from P and travels
and q are constant. If Raju increased his daily
south at 30 km/hr for 48 minutes. At that instant,
production from 10 units to 15 units, his daily
he is 74 km from A. When does B start from P? 8
production cost would increase by 88 /9%. If he
(A) 9:42 a.m. (B) 9:45:20 a.m.
increased his daily production from 15 units to
(C) 9:48 a.m. (D) 9:51:40 a.m.
20 units, his daily production cost would increase by
12
64 /17%. Assume that there is a high demand for the
46. In an x-metre race, A beats B by 180 m and
item and that Raju can sell whatever he produces.
C beats by 351 m. In the same race, B beats
He wishes to maximize his profit.
C by 198 m. Find x?
(A) 1200 (B) 1000 (C) 1122 (D) 1320 52. Raju’s daily production must be (in units).
(1) 10 (2) 15 (3) 8 (4) 5 (5) 12
47. Towns A and D are 36 km apart. Three friends,
Tarun, Varun and Arun start together from 53. Find the maximum daily profit that Raju can
A towards D. While Arun sets off on foot, Varun obtain (in Rs).
takes Tarun along on his bike and travels at (1) 400 (2) 425 (3) 350
45 km/hr. He drops Tarun at a point C and turns (4) 375 (5) 184
back for Arun. He meets Arun at a point B, and
then turns back towards D. All the three friends Directions for questions 54 to 61: Select the
reached D together. If both Arun and Tarun correct alternative from the given choices.
walked at 5 km/hr, how long do the 3 take to
cover AD? 54. E(y) is a quadratic expression. It has the
2 1 13 minimum value of 1 when y = −3 and E(−2) = 2.
(A) 3 (B) 3 (C) 3 (D) 1
Find E(4).
3 9 15
(1) 40 (2) 45 (3) 50 (4) 55 (5) 60
Quadratic Equations 55. If the equations ax2 + bx + c = 0; bx2+ cx + a = 0
have one root in common, then which of the
48. Three consecutive positive integers are taken in following is definitely true?
descending order. The first, second and the third 3 3
(A) a + b = 3abc
are raised to the first, second and third powers (B) a + b3 + c3 = 3abc
3

respectively. The powers are added and the (C) a3 – b3 – c3 = 3abc


result is multiplied by 3. The square root of the (D) a2 + b2 + c2 = 2ab + 2bc + 2ca

4
56. If α, β and γ are the roots of the equation Directions for questions 64 to 76: Select the
x + 2x – 5x – 6 = 0 and αβ, βγ, γα are the roots of
3 2
correct alternative from the given choices.
3 2
the equation x + px + qx + r = 0; then the value of
p is 1 1
64. If k = + +
(A) 6 (B) –5 (C) 5 (D) –6 10.11.12.13 11. 12. 13. 14
1
57. Let α and β be the roots of a quadratic equation ………………+ , then the value
99.100.101.102
and α – β = 9 and |α| – |β| = 5 then find the
product of roots. of k is
(A) –14 (B) –21 (C) –16 (D) –7 2149 1249
(A) (B)
5616600 5666100
58. What is the minimum value of the square of the 1429 4219
difference of the roots of the quadratic equation (C) (D)
2 5666100 5616600
x – (k + 7) x – (3k – 15) = 0?
(A) 160 (B) –180 (C) –160 (D) 180 65. Find the sum of all the five digit numbers which
leave a remainder of either 4 or 6 when divided
59. A certain number of cups of tea are available for by 8.
Rs.90. If the price of each cup increases by (A) 659100000 (B) 561900000
Rs.1.50, the number that can be bought for the (C) 1318200000 (D) 1237522500
same amount decreases by 10. Find the actual
cost of each cup of tea (in rupees). 66. If the sum of the first n terms of an arithmetic
(A) 3 (B) 3.50 (C) 4 (D) 4.50 progression in 2400 and the sum of next n terms
as 7200, then find the ratio of first term and
Progressions / Series common difference.
(A) 3 : 2 (B) 2 : 1 (C) 1 : 2 (D) 2 : 3
60. Find the number of common terms of the
sequences 24, 29, 34, ….. 474 and 25, 29, 33, th t n −1 + t n + 1
67. If the n term of a series is given by tn = ,
……, 485 2
(1) 20 (2) 21 (3) 25 (4) 24 (5) 23
t 31
then is
61. Nilgiris coffee cost Rs.(110 + 0.2N) per kg on the t 27
Nth day of 2006 where 1 ≤ N ≤ 200. Its price (A) 0 (B) 1
th
remained constant that year from the 200 day. (C) –1 (D) None of these
Coorg coffee cost Rs.(97 + 0.25N) per kg on the
Nth day of 2006 where 1 ≤ N ≤ 365. Find the date 68. Find the sum of the first 40 terms of the series
in 2006 on which the prices of the two varieties of 2 + 3 + 4 + 6 + 8 + 9 + 16 + 12 + . . .
coffee were equal. (A) 621 (B) 221 + 628
20 20
(1) June 25 (2) August 10 (C) 2 + 628 (D) 6 + 629
(3) July 31 (4) August 20
Functions
(5) July 21
G(a) a
Directions for questions 62 and 63: Answer these 69. G(x) is a function satisfying = G  for all
questions based on the given information G(b) b
1
M and N are positive quantities. Let g1 = M and h1 = N. real a, b If G(3) = , find G(9).
9
When x is even, gx = Mhx − 1 and hx = Mgx − 1
When x is odd, gx = Nhx − 1 and hx = Ngx − 1 1 1
(1) (2) (3) 81
81 9
62. Which of the following equals gx + hx when x is even? (4) 9 (5) Cannot be determined
1
x
(1) M(MN) 2 (M + N) 70. G(y) is a function satisfying the following conditions.
1 x
x −1 G(1) = 5400 and ∑ G(y) = X2G(X). where X is a
(2) MN 2 (M + N) y =1
1
x −1 natural number greater than 1. Find G(6).
(3) M(MN) 2 (M + N)
2100 2400 1800
1 (1) (2) (3)
(4) MM (M + N)
2 7 7 7
1
x −2
1500
(4) (5) 240
(5) M(MN) 2 (M + N) 7

1 3 10 2
63. If M = and N = , find the least odd x for which 71. If f(xy) = f(x) + f(y) + f(x) f(y) – and f(6) = ,
4 4 9 3
gx + hx < 0.02? then the value of f(1/6) is
(1) 3 (2) 5 (3) 7 (4) 9 (5) 11 2 9 17 17
(A) (B) (C) (D)
3 8 9 8
5
x 77. Which of the following is not possible?
72. If f(x) = , then the value of f(f(f(f(f(f( 3))))) = (1) At least two horses finished before P.
1+ x
(2) There were three horses between Q and P.
5 4 3 6
(A) (B) (C) (D) (3) T finished last.
26 25 19 37 (4) S came in second.
(5) There were three horses between T and R.
73. If f(x +2) = f(x) + 7 and f(1) = 2; f(2) = 5 then the
ratio of f(150) to f(75) is 78. Suppose S finished in the fourth position. Then
(A) 523 : 261 (B) 253 : 621
which of the following is not possible?
(C) 427 : 673 (D) None of these
(1) P finished first.
(2) One horse finished between Q and R.
Averages
(3) Q came in second
74. Twelve years ago, the average of the ages of the (4) T finished last.
members of a joint family having ten members (5) R came second.
was 25 years. Four years later a member aged
50 years died and a child was born in the family Data Sufficiency
that year. Four years after that, another member
aged 50 years died and another child was born. Directions for questions 79 and 80: Answer these
Find the present average age of the members of questions based on the instruction below.
the family (in years).
(1) 26 (2) 27 (3) 28 (4) 29 (5) 30 In the questions below, each question has two
statements A and B following it. Mark your answer as
75. There are two groups (A & B) of children in a joint
family. There are 3 more children in B as (1) if the question can be answered from A alone but
compared to A. While the average age of the not from B alone.
children in A is 6 years more than that of B, the (2) If the question can be answered from B alone but
combined age of the children in A is less than not from A alone.
that of B. After 4 years, the difference in the (3) if the question can be answered from A alone as
combined ages of group A and group B will well as from B alone.
double. If there are less than 10 children in the (4) if the question can be answered from A and B
family, which of the following can be the average together but not from any of them alone.
age (in years) of the children in B? (5) if the question cannot be answered even from A
(A) 9 (B) 8 (C) 7 (D) 5 and B together.
A certain number of players participated in a
76. The number and the average weights of different tournament, played according to the following
groups of children are given below: rules. The number of players at any stage is
Number Average Weight (Kg.) denoted as N.
M 40 N
(i) if N is even, the players are grouped into pairs.
M+2 45 2
M+2 50 The players in every pair play against each other.
7 60 The resulting winners move on to the next round.
If the average weight of the entire class is 50 kg. (ii) If N is odd one player is allowed to move on to
Find the total number of students. the next round. He is said to be given a bye. The
(A) 20 (B) 21 (C) 22 (D) 23 N−1
remaining N – 1 players are grouped in to
2
Quant Based Reasoning pairs who play against each other. The resulting
winners move on to the next round. The players
Directions for questions 77 and 78: Answer these
who lose are eliminated from the tournament.
questions based on the following information. P, Q, R,
From the rules above, it follows that if there are
S and T are 5 horses. They participated in a race.
N
The following are the rules of the race. N players in a round, then players move on to
2
(1) A person who bets on the winning horse gets N+1
the next round if N is even and players
4 times the bet amount. 2
(2) A person who bets on the horse coming second move on to the next round if N is odd.
gets 3 times of the bet amount. This process continues until the final round,
(3) A person who bets on the horse coming in third which is played between two players. The winner
gets back his amount. in this round is the champion.
(4) Other persons lose their amount.
79. Find the number of matches played by the
Mohan had placed his bets on Q, R and T. champion.
The amounts he bet on Q, R and T were Rs.2000, A. In the first round, there were 169 players.
Rs.4000 and Rs.6000 respectively. He ended up with B. The champion was given a bye only once.
no gain and no loss.

6
80. The number of players in the first round was M Alternative 1: Invest in the mutual funds of PQR Ltd.
where 129 < M < 256. Find M. A rise in the stock market will result in a return of −8%
A. One player received a bye while moving from and a fall will result in a return of 10%.
the third to the fourth round.
B. Only one player received a bye in the entire Alternative 2: Invest in the mutual funds of RQP Ltd.
tournament. A rise in the stock market will result in a return of 5%
and a fall will result in return of −4%.
Directions for questions 81 to 84: Each question is
followed by two statements A and B. Indicate your Alternative 3: Invest in a public sector bank which
responses based on the following directives:
promises a 0.4% return.
Mark (1) if the question can be answered using A
85. Find the greatest assured return for Mohan.
alone but not using B alone.
Mark (2) if the question can be answered using B 2
(1) 0.4% (2) 0.5% (3) %
alone but not using A alone. 3
Mark (3) if the question can be answered using A 5
(4) 0.8% (5) %
and B together, but not using either A or 6
B alone.
Mark (4) if the question cannot be answered even 86. Find the strategy which will maximize the
by using A and B together. guaranteed return to Mohan.
(1) 100% in alternative 3.
81. Class X has 80 students. The average height of (2) Equal investment in each alternative.
the students in it is 140 cm. It has two sections, (3) Investments in alternatives 1 and 2 in the
A and B, with equal number of students in each ratio 1 : 2
section. The average height of A exceeds that of (4) Investments in alternatives 1 and 2 in the
B. Mohan is the tallest in A and Sohan is the ratio 2 : 1
shortest in B. If each of these students is (5) Investments in alternatives 1, 2 and 3 in the
transferred to the other section, the average ratio 2 : 3 : 4
heights of the sections would get interchanged.
Find the height of Mohan. Directions for questions 87 to 140: Select the
(A) The average heights of A and B differ by 2 cm. correct alternative from the given choices.
(B) If Sohan shifted from B to A, the average
heights of the sections would become equal. 87. The price of coffee was increased by 40% but Raj
was willing to increase his expenditure by 12%.
82. A company has to store at least 270π kilolitres of Find by what percentage should he decrease his
water at all times to meet safety and regulatory consumption.
requirements. It is considering having a spherical (A) 10% (B) 15% (C) 20% (D) 25%
tank whose wall thickness is uniform and whose
outer radius is 6 meters for this purpose. Will the 88. The ratio of the populations of cities X, Y and Z in
tank meet the company requirements? 2008 was 3 : 5 : 6. The percentage increases in
(A) When empty, the tank weighs 36000π kg. It is the populations of X, Y and Z from 2008 to 2009
made of a material whose density is 4 gm/cc. were 10%, 12.5% and 15% respectively. Find the
(B) The tank’s inner radius is at least 4.5 metres. percentage increase in their total population from
2008 to 2009.
83. P, Q and R are three integers. Find the maximum (A) 12% (B) 14%
value of PQ + QR + PR. (C) 13% (D) None of these
(A) P = Q ≠ R
(B) P + Q + R = 84 89. If the cost of a ball pen reduces by 20%, Raj can
buy 90 more ball pens for Rs.3600. Find the cost
84. P is a point on AB. Rohit wanted to draw a (in Rs.) of a ball pen.
square ABCD but failed to do so. Why did he fail? (A) 12.5 (B) 10 (C) 15 (D) 20
(A) PC = 3 cm
PD 90. Ashok made a loss of 15% by selling 96 apples
(B) PC =
3 for Rs.2040. How many apples must he sell for
Rs.2600 to make a 30% gain?
Percentage, Profit & Loss (A) 80 (B) 100 (C) 65 (D) 104

Directions for questions 85 and 86: Answer these 91. Three persons A, B and C have their monthly
questions based on the information below. incomes in the ratio 6 : 7 : 8. Their monthly
expenditures are in the ratio 5 : 6 : 10.
Mohan was considering three alternatives for investing The monthly savings of C is 37.5% of his monthly
a certain amount. He wanted to get the maximum income. Find what percent of B’s savings was A’s
possible assured return on his investment. The three savings?
alternative are given below. He could make use of (A) 50% (B) 662/3% (C) 831/5% (D) 87.5%
each completely or partially along with the others.
7
Simple Interest – Compound Interest cricket is 7 more than that of students who like
volleyball. The difference of the number of boys
92. The difference between the simple interest and and girls who like cricket and that of the number
the compound interest for two years on a sum of boys and girls who like volleyball are in the
invested at 16% p.a. is Rs.384. Find the sum ratio 16 : 5. Find the strength of the group.
(in Rs.) (A) 90 (B) 100 (C) 110 (D) 120
(A) 13500 (B) 15000 (C) 14250 (D) 12750
Trigonometry
93. Ramu took a certain loan at Simple Interest in
2000 for a period of 4 years. The rate of interest 100. T1 and T2 are two towers and Raju was on the
was constant throughout the loan period top of the tower T1. He realized that there were
whereas had he cleared the loan after 9 years, two points on the ground such that the angle of
he would have paid Rs.90,000. Where as had elevation of T1’s top from each of those points
he cleared the loan after 12 years, he would was α. The distances from, T2’s bottom to the
have paid Rs.105,000. Find the amount (in Rs.) top of T1 as well as to each of the points was
he paid to clear the loan. 30 feet. The area of the triangle formed by the
(A) 45000 (B) 65000 (C) 70000 (D) 75000 top of T1 and each of the points can be (in feet).
(T2 's bottom and the two points on the ground
Time and Work are collinear)
(A) 225 (B) 960
94. Pipe X can fill a tank in a certain time. It was (C) Both (A) and (B) (D) Neither (A) nor (B)
opened at 12 pm. Due to a leak at the bottom of
the tank, the tank was filled only at 12:50 pm.
101. A ladder has a length of 10 m. It makes an
If the leak can empty the tank in 200 minutes,
angle of 45° with a wall. It touches the wall at a
then find the time (in minutes) in which X can fill
the tank. point P. There are two points on the ground.
(A) 30 (B) 45 (C) 40 (D) 50 The angle of elevation of P from each of these
points is 60°. Find the distance between these
95. 8 men can build a wall, 8 m long in 8 days, points (in m). (The two points on the ground and
working 8 hours a day. Find the number of the bottom of the wall are collinear)
hours per day for which 16 men are required to 7 6 10 6 8 6 4 6
work, to build a wall 16 m long in 16 days. (A) (B) (C) (D)
3 3 3 3
(A) 2 (B) 8 (C) 16 (D) 4
pq + qr + rp
96. 50 men can complete a job in 20 days working 102. Given that cos θ = , where p, q, r
9 hours a day. They started the job. They p 2 + q2 + r 2
worked 9 hours each day for the first x days. are real numbers such that the sum of any two
At the end of x days, 5 men left. The remaining exceeds the third, which of the following is not a
job was completed by the remaining men possible value of θ ?
working 8 hours a day in 12.5 days. Find x. (A) 20° (B) 40° (C) 50° (D) 80°
(A) 12.5 (B) 8 (C) 9 (D) 10
Indices, Logarithms, Surds
Venn diagrams
3x + 2
103. 3 – 92x – 1 = 118098, find the value of 6x + x6.
97. In a class, 30% of the students like tea and 40% (A) 945 (B) 559
of the students like coffee. 20% of the students
(C) 2403 (D) None of these
who like tea also like coffee. Find the
percentage of the studies who like neither tea
104. Find the value of
nor coffee.
1 1 1
(A) 32% (B) 34% (C) 38% (D) 36% + + ... +
17 16 + 16 17 18 17 + 17 18 121 120 + 120 121
98. In a locality, 180 residents watch only Sony TV, 20 7 37 13
(A) (B) (C) (D)
210 residents watch only Star Plus and 15 44 44 15
150 residents watch only Zee TV. 540 residents
watch atleast one of Sony TV and Star Plus. 1 1
105. If x + = 3, find the value of x + 6 .
6
At most 340 residents watch Star Plus.
90 residents watch all the three channels. x x
Find the minimum possible number of students (A) 729 (B) 326 (C) 322 (D) 324
who watch Sony TV and Zee TV but not Star
Plus. 106. If a = 5 – 2 6 and b = 5 + 2 6 , evaluate
(A) 40 (B) 50 (C) 20 (D) 30
a4 + b4
.
99. In a group, 60% of the boys and 50% of the girls a3 + b3
like cricket. 45% of the boys and 55% of the 4901 4799 4801
girls like volleyball. Number of students who like (A) (B) (C) 10 (D)
49 485 485

8
3024
107. If log2log4x = log4log2x, find x. 116. Find the last two digits of 7 .
(A) 16 (B) 8 (C) 8 2 (D) 4 2 (1) 41 (2) 81 (3) 21
(4) 01 (5) 61
108. If 1, log7(4x + 5), log7(4x + 1 – 1) are in arithmetic 117. N is a natural number. Div(N), a function of N, is
progression, which of the following is a possible defined as follows:
value of x? Div(N) = N if N ≤ 9
(A) 1 (B) 1.5 (C) 2 (D) 2.5 = Div(S(N)), otherwise,
where S(N) is the sum of the digits of N.
Inequalities For instance, Div(8) = 8,
Div(625) = Div(6 + 2 + 5) = Div(13) = Div(1 + 3)
109. How many integer values of x exist that do not = Div (4) = 4 etc.
satisfy the inequality (x2 + 4x – 32) (x2 + 2x – 8) > 0? Find the number of positive integer values of N
(A) 7 (B) 8 (C) 6 (D) 4 less than 600, for which Div (N) = 9.
(1) 44 (2) 55 (3) 66
110. Under which of the following conditions the (4) 77 (5) 67
a2 b2
inequation − ≤ a − b is true? 1 1 1 1
b a 118. Find the value of 1+ + + 1+ 2 + 2 +
(A) a ≥ b and ab < 0 (B) a ≤ b and ab > 0 12 22 2 3
(C) Either (a) or (b) (D) None of these …… upto 2005 terms
1 1
(1) 2005 − (2) 2006 −
111. What is the set of values of x for which the 2006 2006
4x 2 − 9x + 8 1 1
inequation 2 > 3 is satisfied (3) 2006 − (4) 2005 −
x − 5x − 6 2005 2005
(A) (–6, –1) ∪ (1,∞) (B) (–∞, –6) ∪ (1, ∞) 1
(5) 2006 −
(C) (–∞, ∞) (D) (–∞, –1) ∪ (6, ∞) 2007

112. If x1, x2, x3, x4, x5 and x6 are positive and x1 x2 x3 119. Set A = {3, 4, …….. 2N + 1, 2N + 2}, where N is a
x4 x5 x6 = 1; then the minimum value of (x1 + 4) natural number. Each odd element in it was
(x2 + 4) (x3 +4) (x4 +4) (x5 +4) (x6 +4) is increased by 3 and each even element in it was
(A) 15625 (B) 1425 increased by 1. P denotes the average of the
(C) 13225 (D) 11025 resulting odd elements and Q denotes the
average of the resulting even elements, then
Operator Based Questions P−Q=
(1) 2 (2) 1 (3) −1
113. Given that a ⊕ b = 4a + 3b + 7ab and a ⊕ a < b (4) N (5) N−1
⊕ b, then which of the following is true?
(A) (a – b) (a + b + 1) < 0 120. X and Y are natural numbers. X is odd and less
(B) (a + b) (a – b + 1) > 0 than 100. Find the number of solutions of
(C) (a – b) (a + b+ 1) > 0 1 1 3
= − .
(D) (a+ b) (a – b + 1) > 0 X 18 Y
(1) 4 (2) 3 (3) 2 (4) 1 (5) 5
 3  3
114. If x @ y = y  4 x +  + x  4 y +  , then the 121. A tournament had 2N + 1 teams – t1, t2, ….. t2N+1
 x  y
where N > 6. Each team had x players where
1 x > 4. The following pairs of teams have a
value of 6 @ is
6 common player : t1 and t2N+1, t2 and t2N, ….. tN
3891 and tN+1. These are the only pairs of teams who
(A) (B) 1 have a common player. Find the total number of
36
4179 1297 players in the 2N + 1 teams.
(C) (D) (1) N(x − 1) + x (2) N(2x − 1) + x
36 36
(3) N(2x − 2) + x (4) N(x + 1) + x
(5) N(x + 2) + x
Numbers
122. A four digit number has the form AABB. It is also
115. Raju wrote the first 50 natural numbers one after a perfect square. How many (A, B) are
another on a black board. He then carried out possible?
the following procedure 49 times. In each (1) 0 (2) 3 (3) 4 (4) 1 (5) 2
instance, he erased two numbers, say p and q
and replaced them by a single number p + q – 1. 123. Let M = 3(3!) + 4(4!) + ….. + 15(15!)
Find the final number left on the board. What is the remainder when M – 15 is divided
(1) 1224 (2) 1275 (3) 1276 by 14! – 2?
(4) 1274 (5) 1226 (A) 14! – 443 (B) 14! – 459
(C) 459 (D) 443
9
124. A number when divided by 5, 6, 7 and 8 leaves is removed from the book. What is the sum of
remainders of 3, 4, 5 and 6 respectively. the pages on this sheet?
How many such 4-digit numbers are there? (A) 400 – 2n (B) 400 – n
(A) 11 (B) 10 (C) 9 (D) 12 (C) 200 + 4n (D) None of these

1 1 1 132. If u + v + w + x + y = 15, what is the maximum


125. If x = 2
+ 2 + 2 + .... , what is the value of value of uvx + uvy + uwx + uwy?
2 3 4
(A) 125 (B) 144 (C) 3125 (D) 243
1 1 1
2
+ 2
+ + .... in terms of x?
3 5 72 133. The remainder when 130110 – 1301 is divided
3x − 1 3x x −1 x by 21 is
(A) (B) – 1 (C) (D) –1
4 4 2 2 (A) 1 (B) 2 (C) 19 (D) 20

134. If a, b c, d are natural numbers such that


49 80 − 42 80 23 < a < b < 40 < c < d < 50, how many values
126. If k = , then
49 79 + 42 79 are possible for the quadruplet (a, b, c, d)?
(A) 0 < k ≤ 1 (B) 1 < k ≤ 4 (A) 4284 (B) 4320 (C) 4200 (D) 4165
(C) 4 < k ≤ 7 (D) k > 7
135. A number N leaves a remainder of 7, 10 and
127. If A = 71421 ………. 98 105 112 ……...189 196, 17 respectively when divided by 15, 21, and 35.
what is the remainder when A is divided by 9? What is the remainder when N is divided by
(A) 1 (B) 3 (C) 7 (D) 5 105?
(A) 73 (B) 59 (C) 52 (D) 55
2 5 1/7 −a + b
128. If –2a + 5b = 7(a b ) and x = , what is 136. How many five digit numbers with digits that are
2 not necessarily distinct are divisible by 6 but not
2 5
the value of 2(x + a) + 5(x – b) ? by 12?
(A) 7 (B) 168 (A) 7500 (B) 7499 (C) 14999 (D) 15000
(C) 3 (D) None of these
137. Which of the following numbers does not divide
129. Which of the following is a rational number? (412 – 1)?
(A) (7 + 4 3 )–50 + (7 – 4 3 )50 (A) 5 (B) 63 (C) 255 (D) 127
(B) (7 – 4 3 )–50 + (7 + 4 3 )50 138. Find the number of factors of 243243 which are
log 7 4 3 multiples of 21.
(C) 4 3 + 4 3
7 log7 (A) 20 (B) 21 (C) 22 (D) 24
(D) None of these
139. What is the remainder when 4911 + 5011 + 5111 +
130. Which of the following is prime? 5211 is divided by 202?
(A) 270 + 1 (B) 296 + 1 (A) 0 (B) 101 (C) 201 (D) 1
160
(C) 2 + 1 (D) None of these
140. Q is the number formed by knocking off all the
131. A 200 page book is formed by using 50 sheets, terminal zeros from 256!. What is the index of
folded in the middle and stapled along the fold, the highest power of 12 that divides Q?
with each sheet providing 4 pages. The pages (A) 126 (B) 97
(C) 96 (D) None of these
are then numbered from 1 to 200. The nth sheet

10

You might also like